ChaseDream
搜索
12下一页
返回列表 发新帖
00:00:00

In Gandania, where the government has a monopoly on tobacco sales, the incidence of smoking-related health problems has risen steadily for the last twenty years. The health secretary recently proposed a series of laws aimed at curtailing tobacco use in Gandania. Profits from tobacco sales, however, account for ten percent of Gandania's annual revenues. Therefore, Gandania cannot afford to institute the proposed laws.

Which of the following, if true, most seriously weakens the argument?

正确答案: A

更多相关帖子

524

帖子

15

好友

4712

积分

ChaseDream

注册时间
2003-03-17
精华
8
解析
查看: 2893|回复: 13
打印 上一主题 下一主题

[考古] 求助CR JJ 13考古

[复制链接]
跳转到指定楼层
楼主
发表于 2011-3-4 19:40:57 | 只看该作者 回帖奖励 |倒序浏览 |阅读模式
In Gandania, where the government has a monopoly on tobacco sales, the incidence of smoking-related health problems has risen steadily for the last twenty years. The health secretary recently proposed a series of laws aimed at curtailing tobacco use in Gandania. Profits from tobacco sales, however, account for ten percent of Gandania’s annual revenues. Therefore, Gandania cannot afford to institute the proposed laws.

Which of the following, if true, most seriously weakens the argument?

A.    All health care in Gandania is government-funded.
B.    Implementing the proposed laws is not likely to cause a significant increase in the amount of tobacco Gandania exports.
C.    The percentage of revenue Gandania receives from tobacco sales has remained steady in recent years.
D.    rofits from tobacco sales far surpass any other single source of revenue for the Gandanian government.
E.    No government official in Gandania has ever previously proposed laws aimed at curtailing tobacco use.

Weaken,答案是A. 但是如果问题换成了Assumption,答案是否会是B?

JJ14就是Assumption,请指教!
收藏收藏 收藏收藏
沙发
发表于 2011-3-4 21:22:41 | 只看该作者
我觉得这个考古靠谱…………  
求狗主人确认
板凳
发表于 2011-3-4 21:34:10 | 只看该作者
话说前两天刚做了这个题来着,等狗主人确定~~~
地板
发表于 2011-3-4 21:35:23 | 只看该作者
B is not a necessary assumption.
5#
 楼主| 发表于 2011-3-4 21:39:10 | 只看该作者
我不是狗主人...刚才在考古区看到别人写的,而JJ里面没有.
只是想问,如果问题不是问Weaken,问Assumption的话,那答案还是A么?
6#
 楼主| 发表于 2011-3-4 21:40:32 | 只看该作者
而看到RC JJ14主人写道他选的MS是B,有什么Export,莫非题目有变?
7#
发表于 2011-3-4 21:49:47 | 只看该作者
A is not a necessary assumption.
8#
发表于 2011-3-4 22:03:24 | 只看该作者
14和这个不是一题啊。。
9#
发表于 2011-3-14 20:38:27 | 只看该作者
如果选项不变,ASSUMPTION我觉得D比较合适
10#
发表于 2011-3-14 21:32:37 | 只看该作者
我同意楼上
然后同等待NN确认答案
您需要登录后才可以回帖 登录 | 立即注册

Mark一下! 看一下! 顶楼主! 感谢分享! 快速回复:

手机版|ChaseDream|GMT+8, 2024-9-17 10:36
京公网安备11010202008513号 京ICP证101109号 京ICP备12012021号

ChaseDream 论坛

© 2003-2023 ChaseDream.com. All Rights Reserved.

返回顶部